K
Khách

Hãy nhập câu hỏi của bạn vào đây, nếu là tài khoản VIP, bạn sẽ được ưu tiên trả lời.

19 tháng 1 2019

Áp dụng bất đẳng thức Cô – si ta có:

Đề kiểm tra 45 phút Đại số 10 Chương 4 có đáp án (Đề 3)

Cộng vế với vế bất phương trình (1), (2), (3) ta được:

Đề kiểm tra 45 phút Đại số 10 Chương 4 có đáp án (Đề 3)

15 tháng 2 2021

thử bài bất :D 

Ta có: \(\dfrac{1}{a^3\left(b+c\right)}+\dfrac{a}{2}+\dfrac{a}{2}+\dfrac{a}{2}+\dfrac{b+c}{4}\ge5\sqrt[5]{\dfrac{1}{a^3\left(b+c\right)}.\dfrac{a^3}{2^3}.\dfrac{\left(b+c\right)}{4}}=\dfrac{5}{2}\) ( AM-GM cho 5 số ) (*)

Hoàn toàn tương tự: 

\(\dfrac{1}{b^3\left(c+a\right)}+\dfrac{b}{2}+\dfrac{b}{2}+\dfrac{b}{2}+\dfrac{c+a}{4}\ge5\sqrt[5]{\dfrac{1}{b^3\left(c+a\right)}.\dfrac{b^3}{2^3}.\dfrac{\left(c+a\right)}{4}}=\dfrac{5}{2}\) (AM-GM cho 5 số) (**)

\(\dfrac{1}{c^3\left(a+b\right)}+\dfrac{c}{2}+\dfrac{c}{2}+\dfrac{c}{2}+\dfrac{a+b}{4}\ge5\sqrt[5]{\dfrac{1}{c^3\left(a+b\right)}.\dfrac{c^3}{2^3}.\dfrac{\left(a+b\right)}{4}}=\dfrac{5}{2}\) (AM-GM cho 5 số) (***)

Cộng (*),(**),(***) vế theo vế ta được:

\(P+\dfrac{3}{2}\left(a+b+c\right)+\dfrac{2\left(a+b+c\right)}{4}\ge\dfrac{15}{2}\) \(\Leftrightarrow P+2\left(a+b+c\right)\ge\dfrac{15}{2}\)

Mà: \(a+b+c\ge3\sqrt[3]{abc}=3\) ( AM-GM 3 số )

Từ đây: \(\Rightarrow P\ge\dfrac{15}{2}-2\left(a+b+c\right)=\dfrac{3}{2}\)

Dấu "=" xảy ra khi a=b=c=1

 

 

 

15 tháng 2 2021

1. \(a^3+b^3+c^3+d^3=2\left(c^3-d^3\right)+c^3+d^3=3c^3-d^3\) :D 

21 tháng 8 2021

(a2+b2+c2)3(a2+b2+c2)3 ≥ 9(a + b + c)

21 tháng 8 2021

(a2+b2+c2)3(a2+b2+c2)3 ≥ 9(a + b + c)

AH
Akai Haruma
Giáo viên
31 tháng 7 2023

Lời giải:
Áp dụng BĐT Bunhiacopxky:

$(a^2+b^2+c^2)(1+1+1)\geq (a+b+c)^2$

$\Rightarrow a^2+b^2+c^2\geq \frac{(a+b+c)^2}{3}$

$\Rightarrow (a^2+b^2+c^2)^3\geq \frac{(a+b+c)^6}{27}$

Áp dụng BĐT Cô-si: $a+b+c\geq 3\sqrt[3]{abc}=3$

$\Rightarrow (a^2+b^2+c^2)^3\geq \frac{(a+b+c)^6}{27}\geq \frac{(a+b+c).3^5}{27}=9(a+b+c)$
Ta có đpcm

Dấu "=" xảy ra khi $a=b=c=1$

14 tháng 7 2019

Làm vô đây đài nhưng làm trog giấy ngắn lắm

1) a # b # c # a, thỏa a/(b-c) + b/(c-a) + c/(a-b) = 0 
<=> a(c-a)(a-b) + b(a-b)(b-c) + c(b-c)(c-a) = 0 
<=> -a(a-b)(a-c) - b(b-a)(b-c) - c(c-a)(c-b) = 0 
<=> a(a-b)(a-c) + b(b-a)(b-c) + c(c-a)(c-b) = 0 (*) 
từ (*) ta thấy a, b, c đối xứng nên không giãm tính tổng quát giả sử: a > b > c 

* Nếu a, b, c đều không âm, giả thiết trên thành a > b > c ≥ 0 
(*) <=> (a-b)(a² - ac - b² + bc) + c(c-a)(c-b) = 0 
<=> (a-b)[(a+b)(a-b) -c(a-b)] + c(c-a)(c-b) = 0 
<=> (a-b)².(a+b-c) + c(a-c)(b-c) = 0 (1*) 

thấy b - c > 0 (do b > c) và a > 0 => a+b-c > 0 => (a-b)².(a+b-c) > 0 và c(a-c)(b-c) ≥ 0 
=> (a-b)².(a+b-c) + c(a-c)(b-c) > 0 mâu thuẩn với (1*) 

Vậy c < 0 (nói chung là trong a, b, c phải có số âm) 

* Nếu cả a, b, c đều không có số dương do giả thiết trên ta có: 0 ≥ a > b > c 

(*) <=> a(a-b)(a-c) + (b-c)(b² - ab - c² + ca) = 0 
<=> a(a-b)(a-c) + (b-c)[(b+c)(b-c) - a(b-c)] = 0 
<=> a(a-b)(a-c) + (b-c)².(b+c-a) = 0 (2*) 

a - b > 0; a - c > 0 => a(a-b)(a-c) ≤ 0 (vì a ≤ 0) 
và b < 0; c - a < 0 => b + c -a < 0 => (b-c)².(b+c-a) < 0 
=> a(a-b)(a-c) + (b-c)².(b+c-a) < 0 mẫu thuẩn với (2*) 

chứng tỏ trong a, b, c phải có số dương 

Tóm lại trong 3 số a, b, c phải có số dương và số âm 

1) a # b # c # a, thỏa a/(b-c) + b/(c-a) + c/(a-b) = 0 
<=> a(c-a)(a-b) + b(a-b)(b-c) + c(b-c)(c-a) = 0 
<=> -a(a-b)(a-c) - b(b-a)(b-c) - c(c-a)(c-b) = 0 
<=> a(a-b)(a-c) + b(b-a)(b-c) + c(c-a)(c-b) = 0 (*) 
từ (*) ta thấy a, b, c đối xứng nên không giãm tính tổng quát giả sử: a > b > c 

* Nếu a, b, c đều không âm, giả thiết trên thành a > b > c ≥ 0 
(*) <=> (a-b)(a² - ac - b² + bc) + c(c-a)(c-b) = 0 
<=> (a-b)[(a+b)(a-b) -c(a-b)] + c(c-a)(c-b) = 0 
<=> (a-b)².(a+b-c) + c(a-c)(b-c) = 0 (1*) 

thấy b - c > 0 (do b > c) và a > 0 => a+b-c > 0 => (a-b)².(a+b-c) > 0 và c(a-c)(b-c) ≥ 0 
=> (a-b)².(a+b-c) + c(a-c)(b-c) > 0 mâu thuẩn với (1*) 

Vậy c < 0 (nói chung là trong a, b, c phải có số âm) 

* Nếu cả a, b, c đều không có số dương do giả thiết trên ta có: 0 ≥ a > b > c 

(*) <=> a(a-b)(a-c) + (b-c)(b² - ab - c² + ca) = 0 
<=> a(a-b)(a-c) + (b-c)[(b+c)(b-c) - a(b-c)] = 0 
<=> a(a-b)(a-c) + (b-c)².(b+c-a) = 0 (2*) 

a - b > 0; a - c > 0 => a(a-b)(a-c) ≤ 0 (vì a ≤ 0) 
và b < 0; c - a < 0 => b + c -a < 0 => (b-c)².(b+c-a) < 0 
=> a(a-b)(a-c) + (b-c)².(b+c-a) < 0 mẫu thuẩn với (2*) 

chứng tỏ trong a, b, c phải có số dương 

Tóm lại trong 3 số a, b, c phải có số dương và số âm

Tk mk nha

8 tháng 2 2019

\(\frac{a}{2b+a}+\frac{b}{2c+b}+\frac{c}{2a+c}=\frac{a^2}{2ab+a^2}+\frac{b^2}{2bc+b^2}+\frac{c^2}{2ca+c^2}\)

\(\ge\frac{\left(a+b+c\right)^2}{2ab+a^2+2bc+b^2+2ca+c^2}=\frac{\left(a+b+c\right)^2}{\left(a+b+c\right)^2}=1\)

Dấu "=" xảy ra khi \(a=b=c=1\)

8 tháng 2 2019

bạn giải thích rõ hơn cho mình về xét dấu = xảy ra đc k?

12 tháng 5 2023

Đặt \(P=\dfrac{1}{a^3\left(b+c\right)}+\dfrac{1}{b^3\left(c+a\right)}+\dfrac{1}{c^3\left(a+b\right)}\)

\(P=\dfrac{\left(abc\right)^2}{a^3\left(b+c\right)}+\dfrac{\left(abc\right)^2}{b^3\left(c+a\right)}+\dfrac{\left(abc\right)^2}{c^3\left(a+b\right)}\)

\(P=\dfrac{\left(bc\right)^2}{a\left(b+c\right)}+\dfrac{\left(ca\right)^2}{b\left(c+a\right)}+\dfrac{\left(ab\right)^2}{c\left(a+b\right)}\)

\(P\ge\dfrac{\left(bc+ca+ab\right)^2}{a\left(b+c\right)+b\left(c+a\right)+c\left(a+b\right)}\) (BĐT B.C.S)

\(=\dfrac{ab+bc+ca}{2}\) \(\ge\dfrac{3\sqrt[3]{abbcca}}{2}=\dfrac{3}{2}\) (do \(abc=1\)).

ĐTXR \(\Leftrightarrow a=b=c=1\)

NV
17 tháng 4 2022

Ta có:

\(\left(a+b\right)\left(b+c\right)\left(c+a\right)=\left(a+b+c\right)\left(ab+bc+ca\right)-abc\)

\(=\left(a+b+c\right)\left(ab+bc+ca\right)-\sqrt[3]{abc}.\sqrt[3]{ab.bc.ca}\)

\(\ge\left(a+b+c\right)\left(ab+bc+ca\right)-\dfrac{1}{3}\left(a+b+c\right).\dfrac{1}{3}\left(ab+bc+ca\right)\)

\(=\dfrac{8}{9}\left(a+b+c\right)\left(ab+bc+ca\right)\)

Do đó:

\(\left(a+b\right)\left(b+c\right)\left(c+a\right)\ge\dfrac{8}{9}.3.\left(a+b+c\right)\ge\dfrac{8}{3}\sqrt{3\left(ab+bc+ca\right)}=8\) (đpcm)

Dấu "=" xảy ra khi \(a=b=c=1\)

a+b>=2căn ab

b+c>=2*căn bc

a+c>=2*căn ac

=>(a+b)(b+c)(a+c)>=2*2*2*căn ab*bc*ac=8

20 tháng 10 2019

a, \(\left(a+b+c\right)^2=3\left(ab+bc+ac\right)\Leftrightarrow a^2+b^2+c^2+2ab+2bc+2ac=3\left(ab+bc+ac\right)\)

\(\Leftrightarrow a^2+b^2+c^2-ab-bc-ac=0\Leftrightarrow\left(a-b\right)^2+\left(b-c\right)^2+\left(c-a\right)^2=0\)

=> a=b=c

20 tháng 10 2019

b, \(0=\left(a+b+c\right)^3=a^3+b^3+c^3+6abc+3a^2b+3ab^2+3b^2c+3bc^2+3c^2a+3ca^2\)

\(=a^3+b^3+c^3+6abc+3ab\left(a+b\right)+3bc\left(b+c\right)+3ac\left(a+c\right)\)

\(=a^3+b^3+c^3+6abc-3abc-3abc-3abc\)

\(\Rightarrow a^3+b^3+c^3=3abc\)